Stack Exchange Network

Stack Exchange network consists of 183 Q&A communities including Stack Overflow , the largest, most trusted online community for developers to learn, share their knowledge, and build their careers.

Q&A for work

Connect and share knowledge within a single location that is structured and easy to search.

How to select a dissertation committee member wisely?

I'm currently developing my dissertation proposal, and am in the process of choosing my committee members. I have heard that one should carefully choose their committee members, since they ultimately judge if and when your PhD work is done. As far as I can tell, a committee member should at least have some expertise in my research topic. I'm sure there's more to it than that, but I want to know what other qualities should I look out for? What qualities in a committee member should I avoid? I imagine these other qualities are subtle and difficult to judge at first. Nonetheless, how do I know if they are a good fit for the success of my PhD?

  • thesis-committee

ff524's user avatar

2 Answers 2

First, ask your adviser . He/she likely knows more about most of your department faculty than you do. You don't have to do exactly as you're told. But if you don't, this should be an intentional choice on your part, and you should have a good reason for that choice. Beyond that, I see at least 3 broad areas to consider. You want committee members who will

  • strengthen your professional network : introduce you to potential collaborators, and possibly help with your search for a postdoc or tenure track position; and/or write a letter of recommendation for you
  • give valuable feedback on your work : you think they'll actually read your thesis (you might be surprised how uncommon this is), and might have something constructive to say
  • be easy to work with in the defense process: likely to be flexible on the date of your defense, and likely to sign off on your dissertation without demanding lots of changes (fitting into 4 schedules besides your own can be a nightmare; it's nice to have a few committee members who are easy to work with on this)

Which of these contributions you value most will depend on what you're hoping to do after your PhD. If you're looking to move into industry, many of your professors' contacts may be less valuable to you than if you hope to stay in academia. Do you plan to stay research active? In the field of your dissertation? Practically, you may have limited options. At the very least, you should weigh 1, 2, and 3, and estimate how you think each candidate will contribute in each area.

Dan C's user avatar

  • 13 Only one thing I would add to this excellent answer: At least one committee member should take you out of your professional comfort zone. Do not choose committee members only from your subfield, your department, or even your university. Part of defending your research, especially if you are continuing into academia, is being able to explain your work and defend its importance to people outside your narrow academic circles. –  JeffE Commented Oct 13, 2012 at 16:00
  • @JeffE: One caveat to the suggestion of an external reviewer—try to avoid someone who's geographically far away. Scheduling committee meetings is hard enough as it is; it's exponentially more difficult if you have to organize them around the schedule of someone who needs to fly in for the meetings! –  aeismail Commented Sep 11, 2014 at 8:08
  • 1 @aeismail: That's what Skype is for. I've been in more than one defense where the committee was spread across multiple continents. –  JeffE Commented Sep 11, 2014 at 10:59

This is certainly a very important question. Here is the make up of my committee and its pitfalls. This isn't the best committee but has its advantages. You could easily extrapolate from my experience:

Adviser/Committee member 1:

  • I have a fantastic adviser who always stands by me and supports my work. He essentially fights for me if things go awry.
  • He is also the PI of the project I am working on so my successful graduation and publications do interest him.

Co-adviser/Collaborator/Committee member 2/Extradepartmental committee member:

  • He champions the the ideas behind my research and has done so for the last 20+ years. So there is no problem that I'll get support from him

Committee member 3:

  • Doesn't have anything invested in my research but since it is generally related to his work, he is on board.
  • He is sitting on the fence as far as criticism goes.

Committee member 4:

  • He doesn't like my work since his adviser didn't like this work and its implications.
  • However, since I have a generally pro-me committee, I should be alright but not without breaking a sweat.
  • If I convince him of the merit of my work, I'll have no problem in the future convincing any other detractors See comment by JeffE .

Community's user avatar

You must log in to answer this question.

Not the answer you're looking for browse other questions tagged phd thesis-committee ..

  • Featured on Meta
  • Bringing clarity to status tag usage on meta sites
  • We've made changes to our Terms of Service & Privacy Policy - July 2024
  • Announcing a change to the data-dump process

Hot Network Questions

  • 80 or 90ies Anime, something about a submarine and apocalypse?
  • Meaning of capacitor "x2" symbol on data sheet schematic
  • Is it possible for a company to dilute my shares to the point they are insignificant
  • Can the speed of light inhibit the synchronisation of a power grid?
  • Parallel use of gerund phrases and noun phrases
  • Who said "If you don't do politics, politics will do you"?
  • One IO to control two LEDs. When one is lit, the other is not
  • Is it OK to use the same field in the database to store both a percentage rate and a fixed money fee?
  • What Christian ideas are found in the New Testament that are not found in the Old Testament?
  • python stats.spearmanr and R cor.test(method='spearman') don't return the same p-value?
  • Can you successfully substitute pickled onions for baby onions in Coq Au Vin?
  • What is the default font of report documentclass?
  • Book about a colony ship making an unscheduled stop in a star system with no habitable planets
  • Explaining Arithmetic Progression
  • Why do combinatorists care about Kazhdan–Lusztig polynomials?
  • Home water pressure higher than city water pressure?
  • Is the Shroud of Turin about 2000 years old?
  • Why is global state hard to test? Doesn't setting the global state at the beginning of each test solve the problem?
  • Sun rise on Venus from East or West (North as North Eclliptic Pole)
  • How does quantum field theory affect the ontological tenability of composite objects?
  • Adding guard GND to a battery based system
  • Should I be worried about this giant crack?
  • Highlight shortest path between two clickable graph vertices
  • Which point on a hyperbola is closest to a circle

dissertations committee members

4.8.1 Doctoral Degrees, Dissertations & Dissertation Reading Committees: Policy

Main navigation.

  • 1. Doctoral Dissertation Reading Committee
  • 2. Dissertation Preparation and Submission
  • 3. Certificate of Final Reading
  • 4. Deadlines

Related Content

Last updated on: Monday, March 7, 2022

Completion of a satisfactory dissertation is a university requirement for conferral of a doctoral degree. Policy and procedures for presentation, review and approval of the dissertation are included here.

Submission of an approved doctoral dissertation to the degree program and the Committee on Graduate Studies is required for the PhD and JSD degrees. The doctoral dissertation is expected to be an original contribution to scholarship or scientific knowledge, to exemplify the highest standards of the discipline, and to be of lasting value to the intellectual community. Every doctoral dissertation is read and approved by members of the Stanford faculty to ensure that standards for programmtic and university quality are met. Standards for professional presentation of doctoral work have been established by the Committee on Graduate Studies.

An approved doctoral dissertation is required for the PhD and JSD degrees. Every doctoral dissertation is read and approved by the three members of the student’s doctoral dissertation reading committee.

Authority: 

  • Committee on Graduate Studies (policy)
  • Office of the Registrar  via Stanford Services & Support (implementation)
  • Degree Program Office (implementation)
  • Office of the Vice Provost for Graduate Education  (exceptions)

Applicability: 

PhD & JSD students and programs.

Related Pages: 

4.8.2 Doctoral Degrees, Dissertations & Dissertation Reading Committees: Implementation

1. Doctoral Dissertation Reading Committee

The doctoral dissertation reading committee consists of the principal dissertation advisor and, typically, two other readers. The doctoral dissertation reading committee must have three members and may not have more than five members. At least one member must be from the student’s degree program. Normally, all committee members are members of the Stanford University Academic Council or are emeritus Academic Council members; the principal dissertation advisor must be an Academic Council member. Professors who have recently become emeritus and have been recalled to active duty may serve as principal dissertation advisors, though they are no longer current members of the Academic Council.

A non-Academic Council member (including former Academic Council members) may replace only one of three required members of dissertation reading committees. However, emeritus faculty, whether recalled to active duty or not, count as an Academic Council member on dissertation reading and oral defense committees (clarified by the Committee on Graduate Studies in 2011; see SenD#6535).

The reading committee, as proposed by the student and agreed to by the prospective members, is endorsed by the chair of the major department on the  Doctoral Dissertation Reading Committee form . This form must be submitted before approval of Terminal Graduate Registration (TGR) status or before scheduling a university oral examination that is a defense of the dissertation. The reading committee may be appointed earlier, according to the degree program timetable for doctoral programs. All subsequent changes to the reading committee must be approved by the chair of the major department. The reading committee must conform to university requirements at the time of degree conferral.

Principal Dissertation Advisors and Co-Advisors

Any member of the Academic Council may serve as the principal dissertation advisor. A non-Academic Council member, former Academic Council member, or emeritus Academic Council member may serve as co-advisor with the appointment of a principal dissertation advisor who is currently on the Academic Council. This is to ensure representation for the student in the degree program by someone playing a major advisor role in completion of the dissertation. Professors who became emeritus within two years of the student’s anticipated degree completion and who have been recalled to active duty may serve as principal dissertation advisors, though they are no longer current members of the Academic Council. The reading committee must conform to university requirements at the time of degree conferral.

Requests for further exceptions to the requirement that the principal dissertation advisor be a current member of the Academic Council, for example for recently retired emeritus professors who are still actively engaged on campus, but not recalled to active duty, will be reviewed by the Office of the Vice Provost for Graduate Education.

At their discretion, students may request the appointment of co-advisors who are both members of the Academic Council.

Non-Academic Council Dissertation Reading Committee Membership

The student's department chair or faculty director of graduate studies may, in some cases, approve the appointment of a reader who is not a current or emeritus member of the Academic Council, if that person is particularly well qualified to consult on the dissertation topic and holds a PhD or equivalent foreign degree, via the  Petition for Non-Academic Council Doctoral Committee Members . Former Stanford Academic Council members and non-Academic Council members may thus, on occasion, serve on a reading committee. However, the majority of the examiners must be current or emeritus Academic Council members. More specifically: 

  • If the dissertation reading committee has three or four members, only one non-Academic Council member (including former Academic Council members) may be appointed to the dissertation reading committee. 
  • If the reading committee has five members, up to two non-Academic Council members may be appointed to the dissertation reading committee.

Emeritus Stanford faculty, though no longer current members of the Academic Council, count as Academic Council members on dissertation reading committees (see SenD#6535, 2011).

Prospective committee members in the following categories may be approved without submission of a curriculum vitae: former Academic Council member, visiting professor, visiting associate professor, visiting assistant professor, and senior Stanford University officer who holds a PhD but does not have an academic appointment. 

A curriculum vita is required for prospective committee members in the following categories: senior research associate, senior lecturer, consulting professor, consulting associate professor, consulting assistant professor, acting professor, acting associate professor, acting assistant professor, senior fellow of the Hoover Institution, members of the professoriate at other universities, and distinguished scholars who may currently hold no academic title. The curriculum vita should include a summary of education, professional experience, publications, and academic or other honors.

Exceptions for individuals whose terminal degree is not the PhD or equivalent foreign degree may be granted by the Office of the Vice Provost for Graduate Education (VPGE). Requests for this exception must be approved and submitted to VPGE by the student’s department chair or faculty director of graduate studies. The prospective committee member’s curriculum vitae and a brief description of their contributions to the student's research should be submitted  via email  to the Office of the Vice Provost for Graduate Education.

Changing Membership

Students may petition to add or remove members of the reading committee or change principal dissertation advisors. The resulting committee must conform to university requirements at the time of degree conferral.

In the rare case where a student’s dissertation research on an approved project is in an advanced stage and the principal dissertation advisor is no longer available, every reasonable effort must be made to appoint a new advisor, usually from the student’s reading committee. This may also require that a new member be added to the reading committee before the draft dissertation is evaluated, to keep the reconstituted committee in compliance with the university requirements for its composition. Advisor changes are made with the  Change of Dissertation Adviser or Reading Committee Member form  (see  GAP 3.3 Academic Advising ).

In the event that a student’s principal dissertation advisor leaves Stanford University or becomes emeritus and has not been recalled to active duty, that advisor may continue to work with the graduate student as a co-advisor and serve on the oral and dissertation reading committees, with the appointment of a principal dissertation advisor who is currently a member of the Academic Council. Professors who have recently become emeritus and have been recalled to active duty may serve as principal dissertation advisor, though they are no longer members of the Academic Council. Requests for further exceptions to the requirement that the principal dissertation advisor be a current member of the Academic Council (for example for recently retired emeritus professors who are still actively engaged on campus) will be reviewed by the Office of the Vice Provost for Graduate Education.

Back to top

2. Dissertation Preparation and Submission

The doctoral dissertation must be an original contribution to scholarship or scientific knowledge and must exemplify the highest standards of the discipline. If it is judged to meet this standard, the dissertation is approved for the degree program by the doctoral dissertation reading committee. Information about dissertation format, references, use of published and co-authored work, as well as copyright is on the Registrar's Office website on Format Requirements for eDissertation .

Approvals should be obtained through the electronic signature process (students may submit email confirmations of dissertation approvals from each member of their committee).

Dissertations should be submitted electronically, following the guidelines in:

  • Directions for Preparing Doctoral Dissertations for Electronic Submission
  • Directions for Preparing Engineer Theses for Electronic Submission

Previously published dissertations should not be used as a guide for preparation of the manuscript. The signed dissertation copies and accompanying documents must be submitted to the Office of the Registrar on or before the quarterly deadline indicated in the university’s academic calendar. A fee is charged for the microfilming and binding of the dissertation copies. 

Students are required to either be enrolled full-time or on Graduation Quarter in the term they submit the dissertation (see  GAP 3.1 Registration, Enrollment, and Academic Progress ). The period between the last day of final exams of one term and the first day of the subsequent term is considered an extension of the earlier term. At the time the dissertation is submitted, an Application to Graduate must be on file (filed in Axess), all of the degree program requirements must be complete, and candidacy must be valid through the term of degree conferral.

Dissertations in a Language Other than English 

Dissertations must be in English. Exceptions to permit dissertations in a language other than English are granted by the school dean upon a written request from the chair of the student’s major department.  The student is required to submit directly to the Student Services Center a paper copy of the approval letter (or email message chain) from the school dean.  Approval for writing a dissertation in another language is normally granted only in cases where the other language or literature in that language is also the subject of the discipline. Approval is routinely granted for dissertations in the Division of Literatures, Cultures, and Languages, within degree program specifications. Dissertations written in another language must include an extended summary in English (usually 15-20 pages in length). 

When submitting electronically a dissertation in a language other than English, the student should upload the English summary as a supplemental file.  When submitting such a dissertation on paper, the student is required to submit the abstract for ProQuest in English.

3. Certificate of Final Reading

One reading committee member, who must be a current member of the Academic Council, reads the dissertation in its final form and certifies on the  Certificate of Final Reading  that degree program and university specifications, described below, have been met. Typically, the principal dissertation advisor serves as final reader though another member of the committee who is a current Academic Council member may provide the final signature.

  • All suggested changes have been taken into account and incorporated into the manuscript where appropriate. 
  • If the manuscript includes joint group research, the student's contribution is clearly explained in an introduction.
  • Format complies with university requirements.
  • If previously published materials are included in the dissertation, publication sources are indicated, written permission has been obtained for copyrighted materials, and all of the dissertation format requirements have been met.
  • The dissertation is ready-for-publication in appearance and ready for microfilming and binding.

4. Deadlines

The deadline for submission of dissertations for degree conferral in each term is specified by the  university academic calendar . The final dissertation must be submitted to the Office of the Registrar on or before the quarterly deadlines if degree conferral is desired. 

Some degree programs may set earlier deadlines for the submission of dissertations.

Dissertation deadlines are strictly enforced and no exceptions are made. Students are strongly encouraged to submit their dissertations at least two weeks prior to the deadline to ensure that all requirements can be met in time for the conferral of the degree.

Related Policies

  • GAP 3.1 Registration, Enrollment, and Academic Progress
  • GAP 3.3 Academic Advising
  • GAP 4.7 Doctoral Degrees, University Oral Examinations & Committees

Related Student Services Sections

  • Doctor of Philosophy: Dissertation

Related Information and Forms

  • Doctoral Dissertation Reading Committee form
  • Petition for Non-Academic Council Doctoral Committee Members
  • General Information on Dissertation and Thesis Submission
  • Directions for Preparing Engineer Theses for Electronic Submission
  • Certificate of Final Reading of Dissertation
  • Doctoral Dissertation Agreement Form - UMI/ProQuest
  • Change of Dissertation Adviser or Reading Committee Member
  • Application to Graduate  (in  Axess )
  • Request for Statement of Completion
  • Format Requirements for eDissertation

Prospective Students

  • Current Students
  • Staff Directory

My UNC Charlotte

Campus events.

  • About UNC Charlotte
  • Campus Life
  • Graduate Admissions

Faculty and Staff

  • Human Resources
  • Auxiliary Services
  • Inside UNC Charlotte
  • Academic Affairs
  • Financial Aid
  • Student Health Center

Alumni and Friends

  • Alumni Association
  • Advancement
  • Make a Gift
  • Thesis and Dissertation

Forming Your Committee

Students should not schedule the proposal defense prior to their committee being finalized and their appointment form being approved by the Graduate School.

It is necessary to have the form approved in advance of the proposal defense, as there are instances in which committee members are not approved (for example, if someone is listed as the Graduate Faculty Representative who the Graduate School does not deem  qualified to serve in this capacity).

The Graduate School's requirements for everything from committee formation to graduation clearance can be found under the Current Students tab on the Graduate School website. 

Composition of the Doctoral Committee: Roles and Responsibilities

The Graduate School requires that doctoral committees consist of no less than four members. These four members must be regular members of the Graduate Faculty or must be granted an exception by the Dean of the Graduate School.  All committees must include a chair and a Graduate Faculty Representative. Assistant Professors are usually not approved to serve as chair unless they have served as a committee member first. Exceptions are granted on a case-by-case basis. 

Graduate Faculty Representative

The primary role of the Graduate Faculty Representative is to ensure that the student is treated fairly and that Graduate School policies are upheld. Expertise in the student's area of research is not a requirement. The Graduate Faculty Representative's responsibilities are explained in greater detail here . Assistant Professors are not eligible to serve as Graduate Faculty Representative. 

The requirement to include an outside member on all dissertation committees is not uncommon among institutions of higher education and is in keeping with best practices in doctoral support. 

Committee Members

Committee members are often chosen to provide topic or methodological expertise. Even without contributing their expertise, committee members may be chosen based on faculty with whom the student has a good professional relationship or who could offer a helpful outside perspective. Committee members are generally not as involved as the committee chair in the everyday progression of the dissertation.  Typically, they read the dissertation only in its final form before the defense, although they should be available for consultation throughout the process and may be more closely involved in sections or chapters in which they have particular expertise. 

The committee members and Graduate Faculty Representative will:

  • Approve of the subject matter and methodology of the thesis or dissertation research
  • Review and comment on drafts of the thesis or dissertation prior to submission to The Graduate School
  • Verify, to the best of their ability, the quality of the data collection and evidence, data analysis, and logical reasoning or interpretation in light of the proposal aims
  • Evaluate whether the student’s thesis or dissertation fulfills the requirements of the degree
  • Public Lectures
  • Faculty & Staff Site >>

Forming an Interdisciplinary Dissertation Committee

Doctoral students in interdisciplinary programs face unique challenges in forming dissertation committees. Based on our experience as directors of three such programs (Public Health Genetics, Urban Design and Planning, and Astrobiology), we offer the following suggestions.

Your first challenge

Find the optimal set of members — especially the right chair (or two co-chairs) for your committee. Committee members need to:

  • be the best match for your intellectual interests
  • have the expertise to help you succeed in designing and completing your dissertation
  • be able to help you prepare for your career

In planning for a dissertation, you should consult extensively with faculty members in your program for guidance about:

  • potential research questions
  • planning/timing methodology
  • potential committee members

The role of the committee

The final decision about the appropriate content of your project rests with the dissertation supervisory committee. You should work closely with the committee (especially the chair) to determine your project’s scope and content. The committee will guide your research and should meet regularly with you. Being sure you and your committee agree on what is meant by “regular” meetings is also a good idea. You may find it useful to meet individually with the members and obtain their feedback at several stages of your dissertation process. The interdisciplinary nature of your work may require that feedback at an advanced stage of your dissertation will be provided by the committee in an integrated form. You may want to discuss with your chair how the committee could produce a collective memo integrating their shared feedback.

The composition of dissertation committees

The dissertation supervisory committee must have at least four members, including the chair and the Graduate School representative (GSR). At least three committee members (including the chair and the GSR) must be UW graduate faculty members with an endorsement to chair doctoral committees; a majority of your committee members must be graduate faculty members, identifiable through the  Graduate Faculty Locator .

Committee members should include faculty expertise in your dissertation’s core fields. You might consider having five members, especially if your project involves different disciplines requiring advice and guidance in all areas. Four committee members must attend general and final exams — so having five on your committee provides flexibility if one member cannot attend. However, having more than four committee members may make it more difficult for them to find time to work together.

Selecting a Graduate School representative

You must select the Graduate School representative for your committee by consulting with your chair, other committee members, and/or program directors. The GSR votes and represents the interests of the Graduate School. GSR requirements:

  • be a graduate faculty member
  • have an endorsement to chair doctoral committees
  • no conflict of interest with you or your committee chair

Also, the GSR may not have an official faculty appointment within your committee chair’s department(s) or the department in which your program is housed. This can be challenging for students in interdisciplinary programs. Exceptions to this rule can be made, with appropriate justification, by petition to the dean of the Graduate School.

by Professor Emeritus Melissa Austin, Public Health Genetics; Marina Alberti, professor, Urban Design and Planning; and Woody Sullivan, professor, Astrobiology

  • Skip to Content
  • Catalog Home
  • Institution Home
  • Graduate Catalog /
  • Academic Resources /
  • Advising & Mentoring PhD Students /

The Dissertation Committee

The academic experience is greatly enhanced if faculty members other than the direct advisor are readily and formally available for consultation and discussion with the graduate student. To provide this element of supervision, a dissertation committee must be put in place for the Ph.D. student early in the dissertation stage. The graduate group is responsible for monitoring the progress of the student through the dissertation committee, as follows:

  • A dissertation committee must consist of at least three faculty (including at least two members of the graduate group). While some graduate groups require all members of the dissertation committee be members of the graduate group or affiliated department, others encourage/require appointment of a faculty member from another department to encourage an interdisciplinary perspective. Students should be sure to review the policy about the composition of dissertation committee as they are building their committee.
  • It is required that the dissertation committee meet with the student, as a committee, at least once per year to assess the student’s progress in the program and to provide advice on future work.
  • The committee submits a written report to the graduate group chair, at least once per year, detailing its observations of the student’s progress and its recommendations.
  • The student must be given the opportunity to respond to the committee’s report/recommendation and to append a response to the committee’s report.
  • Copies of the report shall be given to the student and kept by the graduate group.
  • This annual progress report will be used, in part, to determine the mark given for the student’s dissertation status course.

The Graduate Group is responsible for ensuring that the membership of the dissertation committee is recorded in the student’s official University record. The graduate division office at the home school will monitor compliance with this requirement through reports and an annual audit of the official student file.

Advising on Embargo Options

An important point of guidance from the advisor and dissertation committee – that is sometimes overlooked in the later stage of completion -- is to counsel the student on whether to embargo the dissertation. Penn requires open access publication of dissertations in the University’s institutional repository, ScholarlyCommons . Open access publication provides a wide audience, can help to market ideas to potential employers, and can help make plagiarism or theft much easier to detect. The open access dissertation will be available via the internet, including full text searching through search engines like Google. In cases where papers are in press, patents are pending, or where there are other intellectual property concerns, it may be beneficial to delay publication (commonly referred to as an "embargo"). Students should discuss embargo options with their advisor and their dissertation committee who can help to decide whether a delay in publication is necessary or advisable. Refer to the Dissertation Embargo Guidelines for more information about embargo options.

Print Options

Print this page.

The PDF will include all information unique to this page.

A PDF of the entire 2024-25 catalog.

A PDF of the 2024-25 Undergraduate catalog.

A PDF of the 2024-25 Graduate catalog.

Banner

Henley-Putnam's Dissertation or Thesis Committees

  • Getting Started on Finding Your Research Committee

Committee Members Roles and Responsibilities

  • Subject Matter Expert
  • Committee Member

Committee Chair.   The chair schedules the comprehensive exams, delivers feedback and results of the comprehensive exams, acts as an instructor, oversees the production of the thesis/dissertation, communicates feedback from the subject matter expert and committee member, schedules the dissertation defense, meets monthly via Zoom with the student/candidate throughout the research courses, and reviews work for publication quality.  The chair deals directly with the student on the quality of the paper, the presentation, the flow, the sequence, and the conclusions.  

The role of the committee chair includes the following responsibilities:

  • scheduling the comprehensive exams,
  • communicating the grades and feedback from the doctoral comprehensive exam,
  • overseeing the production of the dissertation,
  • managing the timeline and schedule for completion of each phase of the dissertation in the courses.
  • acting as an instructor in the courses,
  • contacting the student/candidate regarding setting and meeting deadlines in the dissertation process,
  • directing the timely and successful completion of each assignment,
  • working directly with the SME and committee member to garner added perspective, feedback, and constructive criticism to strengthen the dissertation,
  • communicating with the student/candidate to convey feedback, insights, added perspective, and constructive commentary provided by the committee member and SME,
  • confirming with the SME that the content of the dissertation is factual and accurate,
  • advising the student on formatting, sequencing, and organizing the thesis/dissertation,
  • ensuring the academic quality of the thesis/dissertation, including each of the assignments in courses.
  • facilitating final approval of the thesis/dissertation by making sure that all committee members sign the approval form, and
  • scheduling and leading the thesis/dissertation oral defense and publication.

Subject Matter Expert (SME).   All members of the committee are subject matter experts (SMEs). The title of this particular member of the committee emphasizes and highlights specific responsibilities within the committee dynamic.  The SME should be in constant contact with the student regarding  content  of the dissertation.  This is the person the student turns to in order to test ideas and conclusions and to ensure the appropriateness, relevance, significance, and accuracy of the dissertation’s content in order to meet university and academic standards.

The SME also certifies the accurate reporting of that material to the chair and determines the factual nature of the work.  The SME knows the subject closely and acts as the student’s sounding board.  The SME does not establish timelines, length of the thesis/dissertation, etc.  The role of SME includes the following responsibilities:

  • consistently consulting with the student/candidate regarding the relevance and significance of the research content,
  • regularly discussing content with the student/candidate to test ideas and conclusions,
  • updating the committee chair about discussions with the student/candidate and about any suggestions or recommendations resulting from those discussions, and
  • confirming the accuracy, appropriateness, relevance, and significance of the research focus and content with the committee chair.

Committee Member.   One additional committee member works for the committee chair.  The committee member advises and assists the committee chair in every aspect of the project.  The committee member interacts directly with the chair, not the student.  This prevents conflicting information from being sent to the student and presents a unified stance during the process.  The chair and the committee member work out all responses presented to the student and resolve any conflicting guidance before the student is contacted.  If conflicts cannot be resolved, the chair makes the ultimate decision.  The chair and the committee member work together constantly, but the chair is the face of the university to the student.  The role of the committee member includes the following responsibilities:

  • interacting with the chair to provide added insight, perspective, and feedback to be shared with the student, and
  • determining responses, suggestions, and constructive criticism that will be shared with the student through the chair.
  • << Previous: Getting Started on Finding Your Research Committee
  • Last Updated: May 22, 2024 11:48 AM
  • URL: https://national.libguides.com/dissertations_Committee

dissertations committee members

Feb. 27, 2023

Selecting your master’s thesis committee members, by karyssa courey: selecting a thesis committee is like completing a puzzle every piece of this process is very valuable to have the right fit for your committee.

puzzle

First off, congratulations on advancing to this stage in your academic career! Meeting the requirements for your thesis is not an easy task but you did it! You deserve to celebrate your achievements, both big and small. 

After you have finish celebrating, it’s now time for you to select committee members for your thesis. A thesis committee is a group of faculty members who provides mentorship for your entire thesis experience.

Before you begin randomly selecting members of your committee, there are a few steps and strategies that will help you pick the right members!

Where do I start?

  • Review the requirements for thesis committees in your department

Rice provides requirements for thesis committees online . For example, a thesis committee for a graduate student in the psychological sciences must have three members (your advisor and two others). At least two committee members must be within your department. Reviewing the requirements will help you understand who can and cannot be on your committee!

  • Breaking down your topic

If you are considering committee members, you likely have a draft of your master’s proposal or at least an idea of what your project will be about. Consider the keywords of your project (i.e., what are five terms that can be used to categorize your proposal?). These keywords are the core of your project and can help you identify faculty that align with your interests and research goals.

  • Connect your topic to faculty research

Now that you have identified your keywords, think about faculty that you know in your department that are knowledgeable in the topics you are studying. You can also search faculty/lab websites or Google Scholar if you are unfamiliar with a faculty member’s research expertise. 

If there are aspects of your master’s proposal that are interdisciplinary, don’t be afraid to search for faculty outside of your department that can provide you with a valuable perspective on your research topic.

Who should I be in communication with?

  • Talk with your advisor

Often, your advisor will have suggestions for potential committee members. Your advisor is one of the few people that will understand both the scope of your research project and know the faculty expertise in your department. Listen to your advisor’s recommendations and suggestions, note any faculty that may be a good fit, and share any ideas that you have based on your search in steps 2 and 3. Don’t feel constricted the recommendations from your advisor either, your peers could also be a resource.

  • Talk with your peers

Older peers in your program often have a great perspective on selecting thesis committee members. Your peers may also have first hand experience with the same faculty members you are considering. E.g., Professor X provides more substantive feedback than Professor Y, so if you want substantive feedback, this is extremely helpful information!

  • Talk with other faculty

Don’t be afraid to schedule meetings with potential faculty members if you want to discuss your thesis. Meeting with faculty is a great way to explain your project, hear their feedback, and gauge their interest.

I’ve talked to everyone on the list; what's next?

  • Email potential committee members

Once you have selected your committee members, it is now time to email them! This might be scary or feel like a daunting step, but remember that the faculty at Rice are here to support you and help cultivate your skills as a researcher. Committee members are your team members, and are here to ideally provide constructive feedback to make your project even better!

When emailing faculty, make sure to use an appropriate tone, provide the title of your thesis, explain your project in a few sentences (or add your abstract), and note any specific reasons that that faculty is a good fit.

For example, if a faculty member has expertise relating to a theory you are applying in your research, make sure to name the theory in your email! The goal is for your potential committee members to have a clear understanding of the scope of your project and connect their expertise to your project.

What should I be mindful of during this process?

  • Handling rejection

If a prospective committee member declines your offer, it’s okay! Do not take it personally or be discouraged! There may be many reasons why a faculty member might decline (e.g., tight on time, perceived lack of fit, or think another faculty member might be a better fit). Do, however, consider other faculty members and consider scheduling a meeting to discuss your thesis project. Meeting with potential committee members can help you understand if they would be a good fit, plus it’s a great opportunity to meet faculty members outside of your courses!

  • Sticking to your timeline

Lastly, keep in mind your deadlines. Different departments have different requirements for scheduling your proposal and declaring your master’s candidacy. For example, in the Psychological Sciences department, a student can propose their master’s without declaring a master's candidacy. However, it is required that committee members are notified at least ten days prior to scheduling your proposal meeting.

Rackham Graduate School: University of Michigan

  • Prospective Students
  • Current Students
  • Postdoctoral Fellows
  • Faculty and Staff
  • Make a Gift

Checklist for Dissertation Chairs

The dissertation committee chair, working with department administrative staff, helps steer the student through the intellectual stages and institutional requirements of doctoral degree work. Advising practices vary from discipline to discipline. However, the outline that follows provides widely applicable guidelines to a chair’s key responsibilities.

The Student-Advisor Relationship

  • Coach the student about your working style and what the student can do to contribute to a good working relationship. (See and refer the student to, How to Get the Mentoring You Want , especially “How to Be a Good Protege,” in Chapter VIII.)
  • If you become aware of significant problems or weaknesses (e.g., in the student’s writing, research-related skills, or personal life), refer the student to appropriate resources .
  • Be aware of students’ individual situations and working style. Occasionally assess how they are doing (e.g., whether they are working well with you and seeking appropriate mentoring from others). See Part Two of Rackham’s, How to Mentor Graduate Students: A Guide for Faculty in a Diverse University .

The Dissertation

  • Help the student to select and refine the dissertation topic, avoiding overly ambitious goals and expectations.
  • Help the student formulate a long-term plan for the research and writing of the dissertation, including a timetable and tentative completion date. Ask the student to revise the plan, if needed.
  • Reach agreement with the student as to how often they will consult with you and submit work for you to critique.
  • Seek a progress report from the student at least once a term.

The Dissertation Committee

  • Assist the student with selecting faculty members to serve on the dissertation committee ( membership guidelines ).
  • (For Co-Chairs) Consult with each other to divide up your supervisory responsibilities, and then inform the student.
  • Make sure everyone on the committee is familiar with the roles of Chair or Co-Chair, cognate, and the other members. How closely and frequently members other than the Chair(s) engage with the student’s work varies, all should be in regular contact.
  • Work with the student to schedule and plan for committee meetings, taking into account the norms of the department or program.
  • If a member of the committee is not responding to the student’s communication or failing to review the draft sections of the dissertation in a reasonable amount of time, coach the student about how to proceed, or intervene directly if the problem is severe. If all efforts fail, encourage the student to consider finding a replacement.
  • Take responsibility for dealing with conflicts among committee members. (e.g., personal conflict and intellectual disputes that create a roadblock for the student).

Supervising Research

  • Emphasize data collection and record keeping.
  • Go over ethical issues, including human subject and animal care protections.
  • Build backup ideas into any research project.
  • Follow the student’s development and make adjustments in assignments.
  • Be aware of conflicts in a research group, and when they arise, take steps to mediate.

Administrative Matters

  • Tell the student to work with the department or program administrative staff to meet Rackham requirements such as filing an up to date Dissertation Committee Form .
  • Where feasible, assist the student in securing funds, such as fellowships, GSIs, GSRAs, research and travel funds.
  • If the Candidate needs to petition Rackham for an extension to the seven-year time limit, provide the necessary support but require the student to develop a plan for completing the degree.

The Oral Defense

  • Tell the student to seek help from the department or program administrative staff about fulfilling Rackham requirements in the final stages of earning the degree.
  • Be sure the student and the committee know that Rackham policy requires that the oral defense must be conducted as a public event, (except for the Committee’s private deliberations either before or after the defense), open to all interested persons.
  • Before the student schedules the oral defense date, be sure the student knows the date in the term of final registration by which the defense must be held. Also, the student must be able to give the entire dissertation to the committee sufficiently in advance of the meeting. This must be at least two weeks before the defense but can be as much as three or four weeks, depending on the Committee.
  • If the student is unable to meet the aforementioned deadline for distributing the dissertation, ask the student to postpone the defense unless you are certain all committee members have critiqued earlier drafts and, therefore, should be able to submit the oral defense evaluation forms on time. It is devastating when a student learns about major concerns shortly before the oral defense because the faculty member hadn’t read the student’s work previously .
  • Prepare the student for the oral defense, in accordance with the traditions of the department and/or the wishes of the committee. (Note: The committee will need to decide how long the defense itself will last; there are no prescribed guidelines.)
  • If you learn that one or more members of the committee have not submitted the Oral Defense Evaluation Form by the Rackham deadline, contact the committee member(s) immediately.
  • If there appear to be serious concerns about the student’s work, advocate for a delay in the oral defense. Dissertations should be approved based on the quality of the work, not because of other pressures (e.g., a job offer contingent upon completion; the expense of registering for a further term, etc.).
  • Before the defense begins, the committee must review all the members’ written evaluations and identify the topics they will raise and their sequence. If any of these activities has not been done in advance, excuse the student and others from the room to do so.
  • At the defense, make sure it’s clear which committee member(s) will sign off on the required revisions.
  • After the defense, submit the completed Oral Defense Examination Form, and, if no revisions or corrections are needed, the Dissertation Completion Form.
  • If revisions and/or corrections are required, make sure the Dissertation Completion Form is submitted as soon as possible after the dissertation has been completed and approved.

Launching the Student’s Career

  • Ask students to do tasks they will need to do after they get into the field.
  • Encourage students to attend professional meetings, and when the two of you attend the same meeting, actively help them to network.
  • Speak honestly to students about their strengths and weaknesses (e.g., not everyone can succeed as a faculty member).
  • If appropriate to your field, call people to help students seek positions and be deliberate and careful about treating them fairly in this regard.
  • Prepare students to consider the full range of career possibilities appropriate to their field.
  • If the student’s dissertation is outstanding, consider nominating it for the ProQuest Distinguished Dissertation Award .

Asking Faculty to Sit on Your Dissertation Committee

Hero Images / Getty Images

  • Tips & Advice
  • Choosing a Graduate Program
  • Admissions Essays
  • Recommendation Letters
  • Medical School Admissions
  • Homework Help
  • Private School
  • College Admissions
  • College Life
  • Business School
  • Distance Learning

The Role of the Dissertation Committee

Choosing a dissertation committee, give some warning, make your intentions known, explain their role, dealing with rejection.

  • Ph.D., Developmental Psychology, Fordham University
  • M.A., Developmental Psychology, Fordham University

Graduate study can best be explained as a series of hurdles. First is getting in. Then comes coursework. Comprehensive exams typically are the culmination of coursework in which you demonstrate that you know your stuff and are ready to begin your dissertation. At this point, you are a doctoral candidate, unofficially known as ABD. If you thought coursework and comps were difficult you’re in for a surprise. Most students find the dissertation process to be the most challenging part of graduate school. It’s how you show that you are an independent scholar capable of generating new knowledge. Your mentor is critical to this process, but your dissertation committee also plays a role in your success.

The mentor is highly invested in the dissertation’s success. The committee serves as an outside consultant, offering a more broad perspective as well as support for the student and mentor. The dissertation committee can serve a checks and balances function that can boost objectivity and ensure that university guidelines are adhered to and that the product is of high quality. Members of the dissertation committee offer guidance in their areas of expertise and supplement the student and mentor’s competencies. For example, a committee member with expertise in specific research methods or statistics can serve as a sounding board and offer guidance that is beyond the mentor’s expertise.

Choosing a helpful dissertation committee isn’t easy. The best committee is composed of faculty who share an interest in the topic, offer diverse and useful areas of expertise, and are collegial. Each committee member should be carefully selected based on the project, what he or she can contribute, and how well he or she gets along with the student and mentor. It’s a delicate balance. You don’t want to argue over every detail yet you need objective advice and someone who will offer insightful, and tough, critiques of your work. Ideally, you should trust each committee member and feel that he or she has your (and your project’s) best interests in mind. Choose committee members whose work you respect, who you respect, and who you like. This is a tall order and finding a handful of faculty who meet these criteria and also have the time to participate on your dissertation committee is a daunting task. It’s likely that not all of your dissertation members will fulfill all of your professional and personal needs but each committee member should serve at least one need.

Work with your mentor to select committee members. As you select potential members, ask your mentor if he or she thinks the professor is a good match to the project. Aside from seeking insight – and making your mentor feel valued – professors talk to each other. If you discuss each choice with your mentor beforehand he is she is likely to mention it to the other professor. Use your mentor’s reaction as an indicator of whether to move forward and approach the potential committee member. You may find that the professor is already aware and may have already implicitly agreed.

At the same time, don’t assume that each professor knows that you’d like them as a committee member. When the time comes, visit each professor with that as your purpose. If you haven’t explained the purpose of the meeting by email then when you enter, sit and explain that the reason you’re asked to meet is to ask the professor to serve on your dissertation committee.

No professor will agree to participate in a project without knowing something about it. Be prepared to explain your project. What are your questions? How will you study them? Discuss your methods. How does this fit with prior work? How does it extend prior work? What will your study contribute to the literature? Pay attention to the professor’s demeanor. How much does he or she want to know? Sometimes a professor might want to know less – pay attention.

In addition to discussing your project, be prepared to explain why you are approaching the professor. What drew you to them? How do you think they will fit? For example, does the professor offer expertise in statistics? What guidance do you seek? Know what the professor does and how they fit in with the committee. Likewise, be prepared to explain why you think they are the best choice. Some faculty might even ask, “Why me? Why not Professor X?” Be prepared to justify your choice. What do you expect expertise-wise? Time-wise? How much or little time and effort will you require? Busy faculty will want to know whether your needs outstrip their time and energy.

If a professor declines your invitation to sit on your dissertation committee, don’t take it personally. Easier said than done but there are many reasons people decide to sit on committees. Try to take the professor’s perspective. Sometimes it’s that they’re too busy. Other times they may not be interested in the project or may have issues with other committee members. It’s not always about you. Participating on a dissertation committee is a lot of work. Sometimes it’s simply too much work given other responsibilities. If they are not able to meet your expectations be grateful that they’re honest. A successful dissertation is the result of a great deal of work on your part but also the support of a helpful committee that has your interests in mind. Be sure that the dissertation committee you build can meet these needs.

  • Avoid These Common Mistakes Students Make in Grad School
  • What is Grad School Like?
  • How to Get Help from Your Professor
  • What Grad Students Can Expect on Their First Day
  • Stop Procrastinating to Complete Your Dissertation
  • 8 Tips to Prepare for Your Comprehensive Examination
  • Graduate School Papers and You
  • What to Do When You Are Accepted to Grad School
  • A Note About Masters and Doctoral Comprehensive Exams
  • You Missed Class: What Do You Do?
  • 8 Tips for Taking Notes from Your Reading
  • Time Management Tips for Graduate Students
  • 6 Tips to Read More in Less Time
  • 6 Reasons to Read Before Class
  • Improve Your Reading Speed and Comprehension With the SQ3R Method
  • What to Do the Summer Before You Start Grad School

About Us arrow_drop_down expand_more

  • News Releases

Our Values arrow_drop_down expand_more

  • Diversity & Inclusion
  • Accessibility
  • Slavery Act Statement

Product Families arrow_drop_down expand_more

dissertations committee members

Content Solutions expand_more

  • Books and Ebooks
  • Dissertations
  • News & Newspapers
  • Primary Sources
  • Streaming Video

Products by Subject expand_more

  • Health & Medicine
  • History & Social Change
  • Interdisciplinary
  • Science & Technology
  • Social Sciences

Popular Products expand_more

  • ProQuest One Academic
  • ProQuest One Business
  • ProQuest One Education
  • ProQuest One Psychology
  • ProQuest Black Studies
  • Ebooks Offers for Libraries

Library Management expand_more

Discovery services expand_more, resource sharing expand_more, course resource lists expand_more, research management expand_more, mobile solutions expand_more.

  • Innovative Mobile

Libraries We Serve expand_more

  • Academic Solutions for universities, colleges, and community colleges of all sizes.
  • Public Solutions for librarians supporting patrons of public libraries.
  • K-12 Solutions for elementary schools, primary schools and high schools.
  • Community College Solutions for community colleges, trade schools and two year programs.
  • Government Solutions for governmental affairs offices, patent examiners, and grants administrators.
  • Corporate Solutions for professionals in the pharmaceutical, legal industries and more.

Solutions For expand_more

  • DEI E-Resources
  • Print to Electronic
  • Reclaiming Your Space
  • Library Management
  • Library Management – Public Libraries
  • Community Engagement
  • Content Discovery
  • Research Repository
  • Digital Preservation
  • Resource Sharing
  • Document Delivery
  • Course Resources

Account Support expand_more

  • Setup and Support
  • Access Questions
  • Renewing a Product
  • Paying an Invoice
  • Get Usage Data: ProQuest
  • Get Usage Data: Alexander Street
  • Submitting Dissertations
  • Idea Exchange
  • ProQuest Status Page

Tools & Resources expand_more

  • Find a Title List
  • Accessibility Documentation
  • Open Access

Browse Collections by Subject expand_more

I want to expand_more.

  • Start my Research
  • Start Text & Data Mining
  • Find Research Funding
  • Keep up with Research News
  • Showcase Research
  • See Upcoming Webinars
  • Contact Support

I’m Interested In expand_more

  • Submitting a Dissertation
  • Purchasing a Dissertation
  • Assembling Course Materials
  • Implementing a Mobile Campus App

Insights expand_more

How text and data mining enables digital literacy in the classroom.

Read about the University of Sydney’s journey to integrate text and data mining (TDM) into its undergraduate courses and incorporate it across disciplines

Meeting Your Needs expand_more

  • Graduate Students
  • Graduate Administrators

Products & Services expand_more

  • ETD Dissemination
  • Dissertation & Theses Global
  • ETD Dashboard

Resources expand_more

  • eLearning Modules
  • Expert Advice Articles

Dissertations News expand_more

  • Top 25 Most-Accessed Dissertations
  • Dissertations Award Winners

Are you a researcher looking for scholarly content? Try searching our platform here...

Language preference

Do you want set this as your default language ?

Connect with ProQuest

Committee selection.

Learn techniques and strategies to help you select a dissertation committee.

dissertations committee members

This expert advice comes from  Sonja Foss and William Waters - authors of   Destination Dissertation: A Traveler's Guide to a Done Dissertation

  • Consult your program handbook to understand the official structure of the dissertation committee and the eligibility of committee members
  • Talk with other students in your department who are further along in the dissertation process about their experiences. Seek out those outside your immediate field who share similar theoretical frameworks or interdisciplinary interests. They may have suggestions regarding faculty you may not have worked with directly or who are outside your department.
  • Seek advice from your advisor. Discuss who you have in mind, and why, and ask for feedback and suggestions. Ideally, you've developed an effective, professional relationship over the course of selecting your topic and writing your dissertation proposal, and she is able to bring that to bear when suggesting committee members.
  • Ask the faculty member to be on your dissertation committee.
  • Make an appointment
  • Email a copy of your one-page dissertation pre-proposal to the faculty member before the meeting. Bring an extra copy to the meeting itself.
  • Be prepared to explain why you'd like the faculty member to be a part of your committee. Does the faculty member provide:
  • Insight into the theoretical framework of your dissertation?
  • Experience working with a particular type of data?
  • Interdisciplinary perspective on your topic?

Working with Your Dissertation Committee Members

  • Understand that while non-advisor faculty members on your dissertation committee may be interested in your topic, they will defer essential questions regarding your dissertation to your advisor.
  • Follow their lead in terms of how much time they'd like to spend discussing your dissertation. Most likely, the initial discussions will be extensive, and then only a notice and review when you're preparing to submit and defend. However, an individual committee member may have a stronger interest in your project, and then you may meet as needed, discussing certain chapters or ideas.
  • Have your chapters and dissertation approved by your advisor before you submit them for review to any other committee members.

Related resources: " Straight Talk about Assembling a Dissertation Committee ," by Mary Renck Jalongo, Ph.D., " Working with Your Committee: A Behind-the-Scenes Look at Sources of Conflict ," by Mary Renck Jalongo, Ph.D.

---------------------------------------------------------------------------------------------------------- 

About the Authors:   Co-authors of   Destination Dissertation: A Traveler’s Guide to a Done Dissertation , Dr. Foss is a professor of Communications at University of Colorado, Denver, and Dr. Waters is an assistant professor of English at University of Houston-Downtown, They are co-directors of Scholar’s Retreat, a program to support progress towards completion of your dissertation, thesis or writing project. 

Additional Resources

Curated advice articles to guide you through dissertations process

Free online learning support to inspire and guide you through the dissertations process

View the top 25 Most-Accessed Dissertations and Theses across all subjects, based upon total PDF downloads.

Learn more about the Worlds Largest Curated Collection of Dissertations and These

Find additional training and informational resources on PQDT

You may also like

How to Pick a Thesis Topic? The Conceptual Conversation

05 September 2014

How to Pick a Thesis Topic? The Conceptual Conversation

Sonja Foss and William Waters, authors of "Destination Dissertation: A Traveler's Guide to a Done Dissertation," share best practices for selecting a thesis topic.

How to Get it Done

15 September 2013

How to Get it Done

Tips for staying focused and organized while writing your dissertation

Writing Your Dissertation or Thesis

Writing Your Dissertation or Thesis

Examine two key strategies for writing a dissertation or thesis more effectively: fast writing and slow revising.

  • Translators
  • Graphic Designers

Solve

Please enter the email address you used for your account. Your sign in information will be sent to your email address after it has been verified.

How To Assemble Your Dissertation Committee

WriteOn

Graduate school is full of complicated assignments and difficult decisions, and most graduate students have developed a process they follow when faced with a daunting task. Since choosing your dissertation committee is one of your most consequential scholarly decisions, you might want to try something besides your standard operating procedure. If it's time to select your dissertation committee but you don't even know where to start, follow the following seven tips to select a stellar dissertation committee.

What is a dissertation committee?

A dissertation committee is a carefully selected group of people who will provide feedback and guidance as you research and compose your dissertation. It is important to pick a balanced group of people for your committee, because these people will help shape your research and ultimately, they will determine whether your dissertation is complete enough to earn you the coveted Ph.D. title.

What does a dissertation committee do?

Dissertation committee members will read sections of your dissertation and provide extensive feedback at various points in your process. They will identify weak arguments and challenge your assertions, with the understanding that you will use their feedback to craft a strong dissertation backed with exceptional research. Make sure to choose people capable of providing constructive feedback, because committee members will expect you to resolve their previous concerns by the next time they review your work.

Many graduate students cringe at the thought of asking faculty members to serve on their dissertation committees, because they erroneously think they're asking potential members for a favor. While serving on your dissertation committee will require members to invest time and effort, it is actually one of their professional responsibilities as faculty members. Keep this in mind as you create your list of possible committee members.

Now that you have a better understanding of the purpose of your dissertation committee, follow these seven essential steps to choose an effective dissertation committee.

1. Review your department's requirements

Consult your departments guidelines to determine how many people you need to select for your dissertation committee and identify any necessary qualifications for committee members. Requirements vary based on your department: Some departments and universities require faculty members to make up a certain portion of the committee while others might require a percentage of the committee to come from external institutions or from your specific industry.

2. Ask for recommendations

  • Ask your advisor if he or she has any recommendations that you should consider for your committee. Your advisor is most likely already invested in you and your research topic, and aside from you, probably has the best understanding of your dissertation topic and your ultimate goals. Your advisor will hopefully have established strong relationships with other faculty members in your department, so he or she should have valuable suggestions for you to consider. Your advisor will probably also serve on your committee (and might even serve as the committee chair), so it will be worthwhile to consider people that you know will work well with your advisor.
  • Ask your fellow graduate students if they can recommend any potential candidates for your dissertation committee. If you have friends who have already selected their dissertation committee, try to learn from their experiences. Ask how they selected their committee members and if they encountered anyone that might be a good fit for you. Friends can also give you tips if a favorite faculty member is going on sabbatical soon or is too busy to serve on any additional committees.

3. Make a list of your ideal committee members

  • Professors and teachers you respect and value
  • Faculty members who have shaped your understanding of your subject matter
  • Industry experts in your field
  • People who will broaden your networking circles: If you plan to seek a career in industry after earning your PhD, select at least one committee member with extensive contacts in your preferred industry (if your department's guidelines allow external members on your committee). Additionally, if you choose well-connected committee members, they might be able to introduce you to future collaborators or suggest research areas you have not yet considered.
  • People with strong communication skills
  • People who will challenge and inspire you

4. Present your list to your advisor and ask for feedback

As you research and write your dissertation, you will seek feedback from your advisor frequently, and you will get some good old-fashioned practice at this during the committee selection process. In Step 2, your advisor recommended a handful of candidates for your committee. Since then, you have added more names to your list, so ask your advisor for feedback on your complete choices. Here are some questions you might want to consider asking:

  • Is there anyone on the list who is difficult to work with and might impede your dissertation progress?
  • Does your list include someone who is already serving on multiple committees and won't be able to give you the time and attention you deserve?
  • Are there any knowledge gaps in your list? In other words, have you included two or three people from the same specialty, which means you probably overlooked including a candidate from another important sector?
  • Does your advisor possess personal or professional information that might lead you to exclude any candidates? Don't be nosy here—you don't need to know the reason, just trust your advisor if he or she tells you that someone you should probably remove someone from your list.

5. Ask your top candidates for a meeting

Email your preferred candidates a brief summary of your research topic and ask if you can schedule an appointment to discuss the possibility of them serving on your dissertation committee. If candidates don't reply to your email within a reasonable time frame, assume that they are too busy to serve on a dissertation committee and remove them from your candidate list. Similarly, if a candidate does not have any availability to meet with you during your selected time frame, that candidate is likely overbooked. You might want to take this as a sign and scratch this person off your list.

6. Meet with your top candidates

If a candidate responded promptly, agreed on a time to meet, and followed through on the meeting, chances are high that he or she is reliable and will be able to fit you into a busy schedule. Remember, you are interviewing this person for to serve on the committee that will have the final say in whether you complete your dissertation and earn your Ph.D. Approach the meeting as if you are interviewing them for a position on your payroll.

Create a list of questions to gauge candidates' knowledge about your topic and to get an idea of their communication styles. Asking the same questions in every meeting will highlight the candidates' similarities and differences, and you can use this information to assemble a compatible group of experts. Ideally, each member of your committee will contribute a different type of knowledge or expertise, so treat these meetings as if you are interviewing to fill a specific role on your team.

After each meeting, take a moment to write down your assessments of the candidate and anything that stands out from the meeting. If a candidate stimulated your mind and offered valuable suggestions during an initial meeting, write that down and make a mark to signify that you definitely want that person on your committee.

In contrast, if you feel an abundance of negativity while meeting with a candidate, it's probably best to eliminate that person from your list of potentials. Choosing who serves on your dissertation committee is one of the most important decisions of your academic career, and you have a better chance of success if you assemble a group of people who challenge and inspire you in positive ways.

7. Ask candidates to join your committee

After you've met with all of your possible candidates, use your post-meeting notes to evaluate your options and create a compatible team. Send a separate email to each potential member and highlight a few reasons that you think he or she will be an ideal choice for your dissertation committee. Explain that you are currently in the process of gathering your dissertation committee and ask candidates to give you an answer by a (reasonable) deadline.

If you decided after a meeting that a candidate was not a good fit for your committee, send a follow up email and thank the person for meeting with you. After expressing your appreciation, state that you have filled all positions on your dissertation committee at this time. If applicable, you might consider asking if you can consult him or her if you have questions while working on your dissertation.

Follow the seven steps above if you want to assemble a dissertation committee composed of experts who will guide you towards a successful dissertation. You're already off to a great start.

Header photo by Luckybusiness .

Want to Get your Dissertation Accepted?

Discover how we've helped doctoral students complete their dissertations and advance their academic careers!

dissertations committee members

Join 200+ Graduated Students

textbook-icon

Get Your Dissertation Accepted On Your Next Submission

Get customized coaching for:.

  • Crafting your proposal,
  • Collecting and analyzing your data, or
  • Preparing your defense.

Trapped in dissertation revisions?

Selecting your dissertation committee members, published by steve tippins on april 12, 2019 april 12, 2019.

Last Updated on: 3rd June 2022, 04:36 am

You have finished all of your classwork and maybe even passed your comprehensive exams. The only thing that you have left to do is write your dissertation. In order to do this, you need to select your dissertation chair and the remainder of your committee members.

While a few institutions that I know of assign committees, even those often allow students to give preferences. What should you think about when choosing your dissertation committee members?

What Do Dissertation Committee Members Do?

Your dissertation committee members serve several roles. First, they are charged with helping you through the dissertation process. While this “help” may seem like hinderance when you receive endless comments and requests for revision, your committee members do this to help you come up with a stronger document.

The second major role your committee plays one can be thought of a gatekeeper. Schools have particular requirements and standards that committees must follow before they can accept a student’s dissertation. Many comments from your committee will pertain to adhering to university requirements.

Committee Chair

You will work most closely with your dissertation committee chair . Be sure to find someone that you can work with. There are stories out there about chairpeople who have bullied those under their charge and used their students as minions to help push the chair’s agenda. Avoid this type of chair.

I believe that you should be looking for someone who wants to serve as a chair and who exhibits real concern for his or her students. Choose someone who has filled the role before and has a track record of helping students finish in a reasonable time period. If your goal is to work in academia, I would also look for someone who has had success publishing and flourishing within an academic setting.

young woman smiling and discussing stuff with dissertation committee

Finally, look for someone who is genuinely interested in your topic and who you get along with.

Other Dissertation Committee Members

Depending upon your institution, you will most likely need to add one to four more members of your committee. The number is not overly important, but the composition is. People usually think of committees being made up of people who bring strength in either the subject matter or the methodology.

If your committee chair is strong in your subject matter then at least one additional committee member should be strong in the methodology you plan to use. Putting your committee together in this way can give you a well-balanced support team.

Read also: Dissertation Committee Request: Sample Email and Guide

Avoid Becoming a Battleground

Just like in life, not everyone in academia likes each other. When dealing in the world of ideas, it seems that it is easy to find people who disagree with each other. Sometimes those disagreements are thoughtful and considerate and sometimes they are not.

Make sure that when you choose committee members that you find people who can work together. You do not want two (or more) committee members using your dissertation to fire barbs at each other. This will be very frustrating and slow you down. It is good to consult with your committee chair about the interpersonal relationships between committee members.

Thank Your Committee

When you finish your dissertation you should be very proud of what you have accomplished.  You have accomplished a great achievement. Remember, without your committee’s help and approval you wouldn’t have made it.

Some people look for and give gifts to their committee members to show their appreciation.  As long as the gifts aren’t too overboard, this is acceptable. At many institutions, there is a tradition of putting your chairperson’s name on your first publication that comes from your dissertation as a thank you. When appropriate, this is a great gesture.

Selecting Your Dissertation Committee Members: Conclusion

It is important to find a dissertation committee that is dedicated to helping you reach your goal. Make sure that you take the time to craft a committee that can provide you the support that you will need and not hinder you. Whatever time you take to create your perfect committee, it will be worth it in the long run.

If you need additional support beyond what your committee can offer you, check out my dissertation coaching and dissertation editing services.

Steve Tippins

Steve Tippins, PhD, has thrived in academia for over thirty years. He continues to love teaching in addition to coaching recent PhD graduates as well as students writing their dissertations. Learn more about his dissertation coaching and career coaching services. Book a Free Consultation with Steve Tippins

Related Posts

female phd student laughing at the laptop

Dissertation

Dissertation memes.

Sometimes you can’t dissertate anymore and you just need to meme. Don’t worry, I’ve got you. Here are some of my favorite dissertation memes that I’ve seen lately. My Favorite Dissertation Memes For when you Read more…

stressed out phd student in front of the computer

Surviving Post Dissertation Stress Disorder

The process of earning a doctorate can be long and stressful – and for some people, it can even be traumatic. This may be hard for those who haven’t been through a doctoral program to Read more…

asian phd student researching on laptop in the library

PhD by Publication

PhD by publication, also known as “PhD by portfolio” or “PhD by published works,” is a relatively new route to completing your dissertation requirements for your doctoral degree. In the traditional dissertation route, you have Read more…

  • Future Students
  • Current Students
  • Faculty/Staff

Stanford GSE

  • Current Students Hub

dissertations committee members

Doctoral handbook

You are here

  • Dissertation Reading Committee

The Doctoral Dissertation Reading Committee consists of three faculty members (the principal dissertation advisor and two other readers) who agree to read a student’s dissertation and serve on the orals committee. All members of an approved reading committee are expected to sign the signature page of the completed dissertation. The reading committee normally serves on the oral exam committee, but not always. At the very least, the primary dissertation advisor and one reader from the reading committee serve on the oral exam committee. The student is responsible for obtaining signatures from advisor and readers before submitting the form to the Doctoral Programs Officer for final processing.

The rules governing the composition of the reading committee are as follows: at least one member of the committee must be from the GSE; the principal dissertation advisor must be on the Stanford Academic Council (AC); and any member of the committee that is not a member of the academic council must be approved by the Area Chair and the Associate Dean of Educational Affairs. In the last case, the Petition for Non-Academic Council Member to Serve on Doctoral Committee form (available from the Doctoral Programs Officer) and a current CV of the proposed member are required. This person must be particularly well qualified to consult on the dissertation topic and hold a PhD or an equivalent foreign degree. Non-AC members may not serve as dissertation advisors, but may serve as a co-advisor along with a member of the AC. Students may only have one non-AC member on the reading committee. The only exception to this rule is if you have more than the three members required for a reading committee. At least two members of the reading committee must be members of the Stanford AC. Reading Committee members must sign the Doctoral Reading Committee form (all forms located on the GSE website under current students>forms). Email confirmations or digital signatures will be accepted.

The reading committee formation, and any subsequent changes to the committee composition, are reviewed and approved by the Associate Dean of Educational Affairs. This signature is obtained by the Doctoral Programs Officer, not the student.

The University requires approval of the Doctoral Dissertation Reading Committee form prior to advancement to Terminal Graduate Registration (TGR) status, or before scheduling a University Oral Examination–whichever comes first in the student’s program. Further instructions for form completion are on the GSE Website.

  • Printer-friendly version

Handbook Contents

  • Timetable for the Doctoral Degree
  • Degree Requirements
  • Registration or Enrollment for Milestone Completion
  • The Graduate Study Program
  • Student Virtual and Teleconference Participation in Hearings
  • First Year (3rd Quarter) Review
  • Second Year (6th Quarter) Review
  • Committee Composition for First- and Second-Year Reviews
  • Advancement to Candidacy
  • Academic Program Revision
  • Dissertation Proposal
  • Dissertation Content
  • University Oral Examination
  • Submitting the Dissertation
  • Registration and Student Statuses
  • Graduate Financial Support
  • GSE Courses
  • Curriculum Studies and Teacher Education (CTE)
  • Developmental and Psychological Sciences (DAPS)
  • Learning Sciences and Technology Design (LSTD)
  • Race, Inequality, and Language in Education (RILE)
  • Social Sciences, Humanities, and Interdisciplinary Policy Studies in Education (SHIPS)
  • Contact Information
  • Stanford University Honor Code
  • Stanford University Fundamental Standard
  • Doctoral Programs Degree Progress Checklist
  • GSE Open Access Policies

PhD students, please contact

Photo of Jeremy Edwards

MA POLS and MA/PP students, please contact

Wesley Horng

EDS, ICE/IEPA, Individually Designed, LDT, MA/JD, MA/MBA students, please contact

Photo of Caroline Stasulat

Stanford Graduate School of Education

482 Galvez Mall Stanford, CA 94305-3096 Tel: (650) 723-2109

  • Contact Admissions
  • GSE Leadership
  • Site Feedback
  • Web Accessibility
  • Career Resources
  • Faculty Open Positions
  • Explore Courses
  • Academic Calendar
  • Office of the Registrar
  • Cubberley Library
  • StanfordWho
  • StanfordYou

Improving lives through learning

dissertations committee members

  • Stanford Home
  • Maps & Directions
  • Search Stanford
  • Emergency Info
  • Terms of Use
  • Non-Discrimination
  • Accessibility

© Stanford University , Stanford , California 94305 .

Graduate Division

  • Committee Composition & Potential Members

A student who is nearing and/or conducting research in preparation for the thesis or dissertation stage is responsible for forming a thesis or dissertation committee. Members of the committee must review the student’s research proposal and indicate their approval by signing Form II. The committee is official once Form II has been approved by Graduate Student Services. The student must keep the committee informed of the scope, plan, and progress of the thesis or dissertation research and manuscript. Note the following policies regarding committee composition:

Committee Size & Potential Members

  • Thesis — three members or more
  • Dissertation — five members or more

Students may conduct an online search to locate potential committee members . The majority of the committee should be faculty from the student’s graduate program. However, exceptions are possible. Emeriti faculty may serve on a committee at the request of the student and the graduate program. Non-graduate faculty (i.e., ad-hoc member) may serve on a committee under special circumstances. If choosing a non-graduate faculty to serve on the committee, attach a request with justification and the individual’s current CV to Form II or submit the request and CV as soon as the student and this individual start working together. Graduate chairs have the privilege of being ex-officio members of all committees in their program.

Committee Chair

The chair is responsible for directing and guiding the student’s research and writing activities. He or she should possess expertise in the student’s chosen research topic. It is the responsibility of the student to select an appropriate topic and to ensure that a faculty member with expertise in the topic is available and willing to serve as chair.

  • Thesis — Chair must be a level 2 or 3 regular or cooperating graduate faculty member in the student’s graduate program.
  • Dissertation — Chair must be a level 3 regular or cooperating graduate faculty member in the student’s graduate program.

Committee Changes

Committee chair : Should a student’s Committee chair retire or resign from the university, if the student is considered ABT/ABD (i.e., eligible to register for Thesis 700 or Dissertation 800), the Committee chair can continue, if willing, to mentor the student through to graduation. If the student is not yet considered ABT/ ABD a new Committee chair would need to be identified to serve as chair.

Committee member : Should a member of a student’s committee retire or resign from the university, if the student is considered ABT/ABD (i.e., eligible to register for Thesis 700 or Dissertation 800), the member can continue, if willing, to serve on the committee as is for up to one year from the time of their departure from the university. If the student needs more than one year to graduate, the Graduate chair should request for the departed committee member to serve as an ad-hoc member (if the program majority requirement is met without them) or nominate the individual for an Affiliate Graduate Faculty (AGF) appointment. If the student is not yet ABT/ABD, the departed faculty member would need to be requested to serve as an ad-hoc member, if willing to continue, nominated for an AGF appointment or replaced.

University Representative (Formerly known as Outside Member)

The University Representative (UR) is an optional member of the thesis committee and a required member of the dissertation committee. The UR fulfills the following functions:

  • The primary responsibility of the UR is ensuring the integrity of the process and appropriate treatment of and participation by the student.

In consultation with his or her committee chair, the student shall identify a faculty member from a different graduate program to serve as the UR. A level 3 cooperating graduate faculty member from another UH campus may not serve as UR. Committee chairs must ensure that the UR is indeed an individual who is at “arm’s length” from themselves and the faculty in the student’s graduate program. Partners and spouses or faculty who may be related or were formerly related, may not serve in roles of chair and UR on the same committee.

Requirements to serve as University Representative:

  • Level 3 member of the graduate faculty on UH Mānoa campus.
  • Experience serving on dissertation committees at UH Mānoa (i.e., completion of service where at least one candidate has graduated).
  • From a different graduate program and at “arm’s length” from the committee chair and faculty in the student’s graduate program.
  • Participation in committee meetings and attendance at the comprehensive exam and dissertation-related presentations (e.g, proposal and defense)
  • If issues and concerns are not resolved with the chair of the committee, the UR should share these issues and concerns with Graduate Division.

Graduate Division 2540 Maile Way Spalding Hall 3rd Floor Honolulu, HI 96822 Tel: (808) 956-8544 Email: [email protected] The University of Hawaiʻi is an Equal Opportunity/Affirmative Action Institution Use of this site implies consent with our Usage Policy | © 2020 Graduate Division | All Rights Reserved

  • Graduate Programs
  • Select a Program
  • Admissions Standards
  • Standardized Exams
  • Documentation Requirements
  • Deadlines & Notification
  • Submitting Your Application
  • Applying for Concurrent Degrees
  • Dual Degrees
  • Multiple Masters or Doctoral Degrees
  • Graduate Certificates
  • Master’s to PhD
  • Readmission
  • Applying as a PBU Student
  • Change in Program
  • UHM Faculty Admission
  • Enrollment Regulations
  • Admissions Requirements
  • English Proficiency
  • English Language Institute (ELI)
  • International Student Services
  • Overseas Advising & Useful Links
  • Admitted Students
  • Cost of Attendance
  • Selected Campus Resources
  • Registration
  • Course Loads & Full-Time Definition
  • Academic Progress
  • Time Allowed for Completion of Degree
  • Leave of Absence
  • Degree Check
  • Commencement
  • Course Applicability
  • Transfer & PBU Credits
  • Credit by Examination
  • Undergraduate Excess Credits
  • Double-Counting Credits
  • Required GDGPA
  • Required Grades
  • Grading Policies
  • Disciplinary Actions
  • Conduct Code
  • Residency Program Requirement
  • Language Requirement
  • Exam Policies
  • Master’s Plan A
  • Master’s Plan B
  • Master’s Plan C
  • Graduate Certificate
  • Change in Degree Requirements
  • Requirements for Concurrent or Sequential Degrees
  • Select Committee Member
  • Format, Language & Content
  • Authorship & Acknowledgement
  • Style Policy
  • Final Defense
  • ProQuest ETD Submission & Publication
  • Survey of Earned Doctorates (SED)
  • Dissertation Written Under Funded Research
  • Manuscript Copyright
  • Remote Participation Resources
  • Student Research Award
  • Compliance & Ethical Standards
  • Co-Authorship
  • Intellectual Property
  • Copyright & Patent
  • Academic Advising
  • Academic Policies
  • Important Information
  • Notice to New GAs
  • Types & Duties
  • Eligibility & Criteria
  • Recruitment & Appointment
  • Clarification of GA workload & compensation
  • Compensation & Tax Withholding
  • Sick and Bereavement Leave
  • Rules & Regulations
  • GA Grievance
  • Frances Davis Award for Excellence in Undergraduate Teaching
  • Record Keeping & Confidentiality
  • Party Responsibilities
  • Informal Resolution
  • Formal Grievance
  • Beyond Grad / Beyond Prof
  • Achievement Scholarships
  • College of Social Sciences Connector
  • East West Center Fellowship and Housing
  • Fellowships & Scholarships
  • Fulbright Annual Competition Calendar
  • Fulbright-Hays Dissertation Research Abroad
  • Graduate Division STAR Fellowships & Scholarships
  • UH Financial Aid Services
  • UH Mānoa International Student Services Scholarships
  • WICHE Program
  • Environment
  • Science & Technology
  • Business & Industry
  • Health & Public Welfare
  • Topics (CFR Indexing Terms)
  • Public Inspection
  • Presidential Documents
  • Document Search
  • Advanced Document Search
  • Public Inspection Search
  • Reader Aids Home
  • Office of the Federal Register Announcements
  • Using FederalRegister.Gov
  • Understanding the Federal Register
  • Recent Site Updates
  • Federal Register & CFR Statistics
  • Videos & Tutorials
  • Developer Resources
  • Government Policy and OFR Procedures
  • Congressional Review
  • My Clipboard
  • My Comments
  • My Subscriptions
  • Sign In / Sign Up
  • Site Feedback
  • Search the Federal Register

This site displays a prototype of a “Web 2.0” version of the daily Federal Register. It is not an official legal edition of the Federal Register, and does not replace the official print version or the official electronic version on GPO’s govinfo.gov.

The documents posted on this site are XML renditions of published Federal Register documents. Each document posted on the site includes a link to the corresponding official PDF file on govinfo.gov. This prototype edition of the daily Federal Register on FederalRegister.gov will remain an unofficial informational resource until the Administrative Committee of the Federal Register (ACFR) issues a regulation granting it official legal status. For complete information about, and access to, our official publications and services, go to About the Federal Register on NARA's archives.gov.

The OFR/GPO partnership is committed to presenting accurate and reliable regulatory information on FederalRegister.gov with the objective of establishing the XML-based Federal Register as an ACFR-sanctioned publication in the future. While every effort has been made to ensure that the material on FederalRegister.gov is accurately displayed, consistent with the official SGML-based PDF version on govinfo.gov, those relying on it for legal research should verify their results against an official edition of the Federal Register. Until the ACFR grants it official status, the XML rendition of the daily Federal Register on FederalRegister.gov does not provide legal notice to the public or judicial notice to the courts.

Design Updates: As part of our ongoing effort to make FederalRegister.gov more accessible and easier to use we've enlarged the space available to the document content and moved all document related data into the utility bar on the left of the document. Read more in our feature announcement .

Administration on Disabilities, The President's Committee for People With Intellectual Disabilities

A Notice by the Community Living Administration on 08/23/2024

This document has been published in the Federal Register . Use the PDF linked in the document sidebar for the official electronic format.

  • Document Details Published Content - Document Details Agencies Department of Health and Human Services Administration for Community Living Document Citation 89 FR 68161 Document Number 2024-18942 Document Type Notice Pages 68161-68162 (2 pages) Publication Date 08/23/2024 Published Content - Document Details
  • View printed version (PDF)
  • Document Dates Published Content - Document Dates Dates Text The meeting will take place on Septmber 26, 2024 from 9:00 a.m. to 4:00 p.m. (EST) and September 27, 2024 from 9:00 a.m. to 3:00 p.m. (EST). Published Content - Document Dates

This table of contents is a navigational tool, processed from the headings within the legal text of Federal Register documents. This repetition of headings to form internal navigation links has no substantive legal effect.

FOR FURTHER INFORMATION CONTACT:

Supplementary information:.

This feature is not available for this document.

Additional information is not currently available for this document.

  • Sharing Enhanced Content - Sharing Shorter Document URL https://www.federalregister.gov/d/2024-18942 Email Email this document to a friend Enhanced Content - Sharing
  • Print this document

This document is also available in the following formats:

More information and documentation can be found in our developer tools pages .

This PDF is the current document as it appeared on Public Inspection on 08/22/2024 at 8:45 am.

It was viewed 0 times while on Public Inspection.

If you are using public inspection listings for legal research, you should verify the contents of the documents against a final, official edition of the Federal Register. Only official editions of the Federal Register provide legal notice of publication to the public and judicial notice to the courts under 44 U.S.C. 1503 & 1507 . Learn more here .

Document headings vary by document type but may contain the following:

  • the agency or agencies that issued and signed a document
  • the number of the CFR title and the number of each part the document amends, proposes to amend, or is directly related to
  • the agency docket number / agency internal file number
  • the RIN which identifies each regulatory action listed in the Unified Agenda of Federal Regulatory and Deregulatory Actions

See the Document Drafting Handbook for more details.

Department of Health and Human Services

Administration for community living.

Administration for Community Living, HHS.

The President's Committee for People with Intellectual Disabilities (PCPID) will host a meeting for its members to discuss the 2024 PCPID Report focused on Home and Community Based Services (HCBS) and discuss emerging issues facing people with intellectual disabilities. All the PCPID meetings, in any format, are open to the public. Members of the public can join in person or virtually. This meeting will be conducted in presentation and discussion format.

The meeting will take place on Septmber 26, 2024 from 9:00 a.m. to 4:00 p.m. (EST) and September 27, 2024 from 9:00 a.m. to 3:00 p.m. (EST).

Comments received by September 13, 2024 will be shared with the PCPID at the September 26-27, 2024 meeting.

Comments: Comments and suggestions may be shared through the following ACL.gov link: https://acl.gov/​form/​pcpid .

In Person/Webinar/Conference Call: The meeting is open to the public and will be hosted at the U.S. Department of Health and Human Services on September 26 and September 27, 2024. The meeting will occur at the Switzer Building Conference Room 1400 located at 330 C Street SW, Washington, DC 20201. Members of the public can observe the meeting in person or virtually. To observe the meeting in person, seating will be available for the first 25 persons to reserve seats due to space limitations. To participate in the meeting virtually, the meeting will be hosted on zoom meeting platform. In order to observe the proceedings in person or virtually, you must register in advance of the meeting at the following link: https://us06web.zoom.us/​meeting/​register/​tZAlceGurzwjH9FLdHPW7YZKOrs3l5GuCDnq .

Mr. David Jones, Director, Office of Intellectual Developmental Disabilities, 330 C Street SW, Switzer Building, Room 1126, Washington, DC 20201. Telephone: 202-795-7367. Fax: 202-795-7334. Email: [email protected] .

Agenda: The Committee will discuss the 2024 PCPID Report focused on Home and Community Based Services as it relates to the areas of direct support professionals, employment, community living, and Federal support programs. And, the committee will begin to examine emerging issues faced by people with intellectual disabilities to be addressed by the Committee. This discussion will help develop a framework for the preparation of the 2025 PCPID Report to the President.

Comments: Stakeholder input is very important to the PCPID. Comments and suggestions especially from people with intellectual disabilities, are welcomed. If there are comments related to HCBS or other areas that you would like to inform the PCPID, please share them through the following ACL.gov link: https://acl.gov/​form/​pcpid .

Background Information on the Committee: The PCPID acts in an advisory capacity to the President and the Secretary of Health and Human Services on a broad range of topics relating to programs, services and support for individuals with intellectual disabilities. The function of PCPID is to: (1) provide such advice concerning intellectual disabilities as the President or the Secretary of Health and Human Services may request; and (2) provide advice to the President and the Secretary of Health and Human Services to promote full participation of people with intellectual disabilities in their communities, such as: (A) expanding educational opportunities; (B) promoting housing opportunities; (C) expanding opportunities for competitive integrated employment; (D) improving accessible transportation options; (E) protecting rights and preventing abuse; and (F) increasing access to assistive and universally designed technologies; and (3) provide advice to the President and the Secretary of Health and Human Services to help advance racial equity and support for people with intellectual disabilities within underserved communities. ( print page 68162)

Statutory Authority: E.O. 14048 , 85 FR 57313 .

Dated: August 14, 2024.

Jennifer Johnson,

Acting Commissioner, Administration on Disabilities.

[ FR Doc. 2024-18942 Filed 8-22-24; 8:45 am]

BILLING CODE 4154-01-P

  • Executive Orders

Reader Aids

Information.

  • About This Site
  • Legal Status
  • Accessibility
  • No Fear Act
  • Continuity Information

WU writing Center logo

  • Doctoral Writing Center
  • Ed.D Dissertation Process

Dissertation Committee

Committee makeup.

dissertations committee members

Dissertation Chair

Role: Primary project manager of the committee. The Chair will have both subject matter and general methodology expertise and will guide the student through the dissertation process. Duration: Begin in year 3 and continue until completion of the dissertation Time Commitment: 24 hours per session with student Selection: Doctoral students will select/approve their Chairs (see below for how to select a Chair)

Committee Member One

Role: Advise students on possible dissertation topics by narrowing down research interests. This committee member will have both subject matter and general methodology expertise. Duration: Begin in year 1 and continue in this role until completion of year 2. In year 3 the Advisor continues to support the dissertation committee in the role of Committee Member # 1 until completion of the dissertation Time Commitment: 2 hours per session in year 1, and 2 and 4 hours per dissertation course in year 3 with student Selection: The Dissertation Director will assign this committee member after matching expertise with student focus.

Committee Member Two

Role: Review dissertation document in preparation for preliminary and final defense. This committee member will have both support subject matter/and general methodology expertise. Duration: Begin in year 3, for courses 3 and 4 (BUS 902 & BUS 903) Time Commitment: 2 hours per course with student Selection: The Dissertation Director/Program Chair will assign this committee member after matching expertise with student focus.

Responsibilities by Roles

The responsibilities of the Chair include:

  • Staying current with dissertation policies and procedures
  • Implementing any changes
  • Advising the candidate from the first dissertation course until the completion of the dissertation and graduation (BUS 901 – BUS 903)
  • Guiding the candidate to complete a work plan for completion of the dissertation in each dissertation course
  • Guiding the candidate toward achieving a high level of technical and ethical quality in the dissertation research
  • Assisting the candidate completing the CITI and in navigating the IRB approval process
  • Assisting the candidate to complete their proposal
  • Providing guidance on the dissertation structure, formatting, content
  • Referring candidate to support functions when necessary – Doctoral Writing Center, Methodology Specialists
  • Guiding the candidate in the selection of methods/procedures for data collection and analysis
  • Preparing the candidate for the defense process

Dissertation Committee Members - Advisor/Reader

All members of the candidate’s committee share responsibility in ensuring that the candidate produces high-quality scholarship. The responsibilities of the Committee Members include:

  • Providing subject matter expertise and guidance
  • Reading drafts and provide meaningful feedback at each defense stage
  • Providing guidance on correct usage of APA
  • Directing student to editors list

Doctoral Candidate

The responsibilities of you as a doctoral candidate include:

  • Proposing a viable project that has collectible data to support conclusions.
  • Managing the doctoral research process, including initiation and continuation of communications with the Dissertation Chair and the Dissertation Committee Members.
  • Completing weekly work plans and contact form in GAP and bi-weekly meetings with the Dissertation Chair.
  • Meeting and abiding by the deadlines in the written and approved work plan and contact form.
  • Completing a successful preliminary defense.
  • Conducting ethical research that adheres to the approved written methodology received by the Institutional Review Board (IRB).
  • Completion of CITI certification
  • Completing a successful final defense.
  • Incorporating any feedback and recommendations from the Dissertation Chair and the Dissertation Committee Members.
  • Having a deliverable, scholarly written, edited, and properly formatted final draft of the dissertation research meets the university’s content and quality standards. (This is the completed Dissertation that must be successfully defended.)
  • Keeping the Dissertation Chair and the Dissertation Committee Members informed of developments as the research study is conceptualized, designed, conducted, and written. A Doctoral Candidate may consult with an additional statistician, Methodologist, or editor, but in no case should any person other than the Doctoral Candidate conduct the work associated with the dissertation research.

Note: If an event occurs that prohibits the dissertation research’s progression and completion; the Doctoral Candidate must communicate with the Dissertation Chair to obtain advice, service, or assistance. If any significant modifications need to be made to the timeline, the Doctoral Candidate must seek approval from the Dissertation Chair and the Dissertation Director.

Selecting Your Dissertation Chair

At the end of the second year, after successful completion of RES 751, doctoral students will select their Dissertation Chair. The Dissertation Chair serves as the committee lead, providing expert support structure in content, methodology and guidance throughout the dissertation process.

After successful completion of RES 751, students will receive an email from Student Services with a link to the Dissertation Committee Selection Form. Students will use this form to identify their first and second choice for Dissertation Chair.

Considerations for Selection

Selecting your Dissertation Chair is a crucial step in the dissertation process and should be done so with careful consideration. For advice on what to consider prior to selection, please see here .

To see a list of the available Dissertation Chairs and their subject matter expertise and qualifications, please see here .

Additional Support

While the students’ Committee Members and Chair will be able to provide substantial support throughout the dissertation process, additional dissertation support is available.

Doctoral Writing Center Specialists

Doctoral Writing Center Specialists are available to assist students from the time they begin their prospectus until the end of your dissertation. The specialists are able to guide the process of writing, organization and revising the dissertation.

The responsibilities of the Doctoral Writing Center Specialists include:

  • Provide suggestions and considerations for the author on organization, mechanics, cohesion, or flow.
  • Focus on the areas that the author has specified the need for.
  • Discuss aspects related to writing, not content.

How to Get in Touch: Book an appointment easily here!

Methodology Specialists

Methodology Specialists provide support to dissertation students if it is determined that a proposed dissertation is sufficiently complex to require more in-depth guidance. Individuals in this role guide dissertation students in selecting the best approach to data collection and analysis.

Roles and responsibilities include but are not limited to:

  • Providing in-depth guidance into both qualitative and quantitative research methodologies
  • Meeting students for 2-hour consultancy sessions (days/times may vary)
  • Proposing the most appropriate research design fit to address a problem statement and research questions or for testing stated hypotheses
  • Data collection and analysis
  • Presentation of the data
  • Conclusions drawn from analyses

How to Get in Touch: Please send an email to [email protected] for a current list of methodologists available.

Common Questions

For a list of responses to common questions regarding the chair selection process, please see here .

Contact Options If you have questions regarding any element of this process, please reach out to:

Dr. Geraldine Goodstone, Dissertation Director – [email protected] Dr. Alex Sherm, Program Chair – [email protected]

IMAGES

  1. Dissertation Committees

    dissertations committee members

  2. Dissertation Committee: Roles, Functions, and How to Choose

    dissertations committee members

  3. (PDF) Dissertations Completed by Author, Title, Committee Chairs and

    dissertations committee members

  4. Dissertation Committees

    dissertations committee members

  5. Dissertation Committee: Roles, Functions, and How to Choose

    dissertations committee members

  6. All You Need to Know About a Dissertation Committee

    dissertations committee members

COMMENTS

  1. Dissertation Committee: Roles, Functions, and How to Choose

    The basic function of your dissertation committee, which typically consists of five members, is to guide you through the process of proposing, writing, and revising your dissertation. Dissertation committee members serve in a mentoring capacity, offering constructive feedback on your writing and research, as well as guiding your revision ...

  2. PDF Dissertation Committee Roles, Responsibilities and Checklist

    Once the dissertation draft is complete, the chair sends it to the member and dean's rep to get feedback. A date for the defense will be set by the chair at least 3 weeks from the date of receipt of the complete dissertation. The committee member will read the . entire . dissertation closely

  3. Roles and Responsibilities of Dissertation Committee

    Committee Chair. The chair schedules the comprehensive exams, delivers feedback and results of the comprehensive exams, acts as an instructor, oversees the production of the thesis/dissertation, communicates feedback from the subject matter expert and committee member, schedules the dissertation defense, meets monthly via Zoom with the student/candidate throughout the research courses, and ...

  4. Guidelines for Dissertation Committee Service » Rackham Graduate School

    Dissertation committees must have at least four members, three of whom are members of the graduate faculty ( see definition above ), and two of whom are from the doctoral candidate's home program. Furthermore, each committee: Must have a sole chair or two co-chairs. Must have a cognate member who is familiar with the standards for doctoral ...

  5. How to select a dissertation committee member wisely?

    13. Only one thing I would add to this excellent answer: At least one committee member should take you out of your professional comfort zone. Do not choose committee members only from your subfield, your department, or even your university. Part of defending your research, especially if you are continuing into academia, is being able to explain ...

  6. 4.8.1 Doctoral Degrees, Dissertations & Dissertation Reading Committees

    3. Certificate of Final Reading. One reading committee member, who must be a current member of the Academic Council, reads the dissertation in its final form and certifies on the Certificate of Final Reading that degree program and university specifications, described below, have been met. Typically, the principal dissertation advisor serves as final reader though another member of the ...

  7. Forming Your Committee

    The committee members and Graduate Faculty Representative will: Approve of the subject matter and methodology of the thesis or dissertation research. Review and comment on drafts of the thesis or dissertation prior to submission to The Graduate School. Verify, to the best of their ability, the quality of the data collection and evidence, data ...

  8. Forming an Interdisciplinary Dissertation Committee

    The dissertation supervisory committee must have at least four members, including the chair and the Graduate School representative (GSR). At least three committee members (including the chair and the GSR) must be UW graduate faculty members with an endorsement to chair doctoral committees; a majority of your committee members must be graduate ...

  9. The Dissertation Committee < University of Pennsylvania

    A dissertation committee must consist of at least three faculty (including at least two members of the graduate group). While some graduate groups require all members of the dissertation committee be members of the graduate group or affiliated department, others encourage/require appointment of a faculty member from another department to ...

  10. Henley-Putnam's Dissertation or Thesis Committees

    Remaining members of the committee may also nominate a replacement. Nominations must be submitted to the associate dean of the Doctorate in Strategic Security in writing no more than 30 days following the loss of the committee member. 2. A committee may elect to remove one of its members by majority vote. A letter describing the reasons for the ...

  11. Committee Members Roles and Responsibilities

    Committee Chair. The chair schedules the comprehensive exams, delivers feedback and results of the comprehensive exams, acts as an instructor, oversees the production of the thesis/dissertation, communicates feedback from the subject matter expert and committee member, schedules the dissertation defense, meets monthly via Zoom with the student/candidate throughout the research courses, and ...

  12. The Role of the Dissertation Committee

    Nevertheless, the committee plays important roles that often benefit the university more than the student. Although every program is different, the dissertation committee plays four roles during a dissertation. Committee members run the gamut from highly involved players to figureheads simply occupying a seat.

  13. Selecting Your Master's Thesis Committee Members

    Older peers in your program often have a great perspective on selecting thesis committee members. Your peers may also have first hand experience with the same faculty members you are considering. E.g., Professor X provides more substantive feedback than Professor Y, so if you want substantive feedback, this is extremely helpful information! ...

  14. Checklist for Dissertation Chairs » Rackham Graduate School: University

    Assist the student with selecting faculty members to serve on the dissertation committee ( membership guidelines ). (For Co-Chairs) Consult with each other to divide up your supervisory responsibilities, and then inform the student. Make sure everyone on the committee is familiar with the roles of Chair or Co-Chair, cognate, and the other members.

  15. Asking Professors to Sit on Your Dissertation Committee

    Members of the dissertation committee offer guidance in their areas of expertise and supplement the student and mentor's competencies. For example, a committee member with expertise in specific research methods or statistics can serve as a sounding board and offer guidance that is beyond the mentor's expertise.

  16. Committee Selection

    Ask the faculty member to be on your dissertation committee. Make an appointment. Email a copy of your one-page dissertation pre-proposal to the faculty member before the meeting. Bring an extra copy to the meeting itself. Be prepared to explain why you'd like the faculty member to be a part of your committee. Does the faculty member provide:

  17. How To Assemble Your Dissertation Committee

    4. Present your list to your advisor and ask for feedback. As you research and write your dissertation, you will seek feedback from your advisor frequently, and you will get some good old-fashioned practice at this during the committee selection process. In Step 2, your advisor recommended a handful of candidates for your committee.

  18. Selecting Your Dissertation Committee Members

    First, they are charged with helping you through the dissertation process. While this "help" may seem like hinderance when you receive endless comments and requests for revision, your committee members do this to help you come up with a stronger document. The second major role your committee plays one can be thought of a gatekeeper.

  19. Dissertation Reading Committee

    The Doctoral Dissertation Reading Committee consists of three faculty members (the principal dissertation advisor and two other readers) who agree to read a student's dissertation and serve on the orals committee. All members of an approved reading committee are expected to sign the signature page of the completed dissertation. The reading committee normally serves on the oral exam committee ...

  20. Dissertation Committees

    Committee Member One. Role: Read the dissertation manuscript and provide suggestions on substantive editoral changes. Will attend Preliminary and Final Defense. Duration: DIS 901, DIS 903 Selection: The Dissertation Coordinator will assign this committee member after matching expertise with student focus in DIS 901.

  21. Committee Composition & Potential Members

    Committee Size & Potential Members. Thesis — three members or more. Dissertation — five members or more. Students may conduct an online search to locate potential committee members. The majority of the committee should be faculty from the student's graduate program. However, exceptions are possible. Emeriti faculty may serve on a ...

  22. Administration on Disabilities, The President's Committee for People

    The President's Committee for People with Intellectual Disabilities (PCPID) will host a meeting for its members to discuss the 2024 PCPID Report focused on Home and Community Based Services (HCBS) and discuss emerging issues facing people with intellectual disabilities. All the PCPID meetings, in any format, are open to the public.

  23. Lancaster County prison committee uses closed meetings, non-disclosure

    Last year, consultants hired by the county and the advisory committee members collaborated on a detailed 136-page report that laid out what operations and programming could be included in the new ...

  24. Dissertation Committees

    Committee Member Two. Role: Review dissertation document in preparation for preliminary and final defense. This committee member will have both support subject matter/and general methodology expertise. Duration: Begin in year 3, for courses 3 and 4 (BUS 902 & BUS 903) Time Commitment: 2 hours per course with student Selection: The Dissertation Director/Program Chair will assign this committee ...

  25. CD61/3

    Statement of Assessed Contributions due from Member States; What We Do. Communicable Disease Prevention, Control, and Elimination; Comprehensive Immunization ... CD61/3 - Election of Three Member States to the Executive Committee on the Expiration of the Periods of Office of Argentina, the Plurinational State of Bolivia, and Jamaica. Download ...